Difference between revisions of "2024 AMC 10A Problems/Problem 21"
m (Protected "2024 AMC 10A Problems/Problem 21" ([Edit=Allow only administrators] (expires 04:59, 8 November 2024 (UTC)) [Move=Allow only administrators] (expires 04:59, 8 November 2024 (UTC))) [cascading]) |
|||
Line 1: | Line 1: | ||
+ | ==Problem== | ||
+ | The numbers, in order, of each row and the numbers, in order, of each column of a <math>5 \times 5</math> array of integers form an arithmetic progression of length <math>5{.}</math> The numbers in positions <math>(5, 5), \,(2,4),\,(4,3),</math> and <math>(3, 1)</math> are <math>0, 48, 16,</math> and <math>12{,}</math> respectively. What number is in position <math>(1, 2)?</math> | ||
+ | <cmath> \begin{bmatrix} . & ? &.&.&. \\ .&.&.&48&.\\ 12&.&.&.&.\\ .&.&16&.&.\\ .&.&.&.&0\end{bmatrix}</cmath> | ||
+ | <math>\textbf{(A) } 19 \qquad \textbf{(B) } 24 \qquad \textbf{(C) } 29 \qquad \textbf{(D) } 34 \qquad \textbf{(E) } 39</math> | ||
+ | ==Solution== |
Revision as of 16:52, 8 November 2024
Problem
The numbers, in order, of each row and the numbers, in order, of each column of a array of integers form an arithmetic progression of length The numbers in positions and are and respectively. What number is in position